Find a function with given condition

Click For Summary
To find a curve passing through point A(2,0) such that the triangle formed by a tangent at point M, the y-axis, and the secant OM is isosceles, the function can be expressed as y=k(x-2), where k is non-zero. The discussion highlights the need to define point M, which has coordinates (x, f(x)), and its position relative to point A. The key relationship involves equating the distances from M to the y-axis and from the origin to the tangent's intersection with the y-axis. This leads to a first-order differential equation that the function f must satisfy, with the initial condition f(2) = 0. The exploration of the problem emphasizes the geometric properties of the triangle and the conditions for isosceles formation.
gruba
Messages
203
Reaction score
1

Homework Statement


Find a curve that passes through point A(2,0) such that the triangle which is defined with a tangent at arbitrary point M, axis Oy and secant \overline{OM} is isosceles. \overline{OM} is the base side of a triangle.

2. The attempt at a solution
Function passes through point A(2,0)\Rightarrow y=k(x-2),k\neq 0.
There are two cases, for k<0 and for k>0.

What is the relation between points A and M because M is not defined?

In which quadrant the point M should be?
 
Physics news on Phys.org
M is an arbitrary point in the curve; its coordinates are (x, f(x)).

I think the assertion is that the distance along the tangent from M to the y-axis is equal to the distance from the origin to the intersection of the tangent and the y-axis. That allows you to write down a first-order differential equation which f must satisfy, and your initial condition comes from the constraint that A lies on the curve (ie f(2) = 0).
 
  • Like
Likes BvU
Question: A clock's minute hand has length 4 and its hour hand has length 3. What is the distance between the tips at the moment when it is increasing most rapidly?(Putnam Exam Question) Answer: Making assumption that both the hands moves at constant angular velocities, the answer is ## \sqrt{7} .## But don't you think this assumption is somewhat doubtful and wrong?

Similar threads

  • · Replies 17 ·
Replies
17
Views
2K
  • · Replies 1 ·
Replies
1
Views
3K
Replies
4
Views
2K
  • · Replies 8 ·
Replies
8
Views
2K
  • · Replies 1 ·
Replies
1
Views
2K
  • · Replies 39 ·
2
Replies
39
Views
6K
  • · Replies 10 ·
Replies
10
Views
3K
  • · Replies 1 ·
Replies
1
Views
1K
  • · Replies 12 ·
Replies
12
Views
2K
  • · Replies 7 ·
Replies
7
Views
2K